Finding polar form of complex number

Click For Summary
To convert the complex numbers -3 and 18 + 4.19i into polar form, the magnitude r is calculated using r = (a^2 + b^2)^(1/2). The angle θ can be derived from both cosine and sine functions, resulting in two potential angles: 127.2 degrees and 52 degrees. The discrepancy arises because both angles yield the same sine value, but differ in cosine, which helps identify the correct angle for the given complex number. The correct angle corresponds to the specific quadrant of the complex number, confirming that it is normal to obtain two different values for θ. Ultimately, determining the appropriate angle is essential for accurate representation in polar form.
astrololo
Messages
200
Reaction score
3

Homework Statement


I have the following complex numbers : -3,18 +4,19i
I must put it in polar form.

Homework Equations


r=(a^2+b^2)^(1/2)
cos x = a/r
sin x = b/r

The Attempt at a Solution



I was able to find with cos x = a/r that the x = 127,20

But when I do it with sin x = b/r I obtain like 52 degrees. I know that I Must obtain 127,20 for BOTH. Why isn't it working ?
 
Physics news on Phys.org
The sin of both those angles are the same, so you must decide which is correct. That is the angle that has the correct a,b values. 52 degrees would be at (3.18, 4.19) and 127.2 degrees is at (-3.18, 4.19).
 
  • Like
Likes astrololo
FactChecker said:
The sin of both those angles are the same, so you must decide which is correct. That is the angle that has the correct a,b values. 52 degrees would be at (3.18, 4.19) and 127.2 degrees is at (-3.18, 4.19).

Oh ok so its normal that I obtain two different values. Ok then, thank you!
 
Yes, There are always two values of \theta in the interval 0 to 2\pi that have the same sin(\theta). But you still have to determine which is correct for the specific problem- the two different values, \theta have different values for cos(\theta).
 
Question: A clock's minute hand has length 4 and its hour hand has length 3. What is the distance between the tips at the moment when it is increasing most rapidly?(Putnam Exam Question) Answer: Making assumption that both the hands moves at constant angular velocities, the answer is ## \sqrt{7} .## But don't you think this assumption is somewhat doubtful and wrong?

Similar threads

  • · Replies 19 ·
Replies
19
Views
2K
Replies
3
Views
1K
  • · Replies 3 ·
Replies
3
Views
2K
  • · Replies 14 ·
Replies
14
Views
2K
  • · Replies 4 ·
Replies
4
Views
3K
  • · Replies 1 ·
Replies
1
Views
2K
  • · Replies 6 ·
Replies
6
Views
2K
  • · Replies 1 ·
Replies
1
Views
2K
  • · Replies 5 ·
Replies
5
Views
2K